Tài liệu Bất đẳng thức Muirhead và một vài áp dụng ppt

8 936 14
Tài liệu Bất đẳng thức Muirhead và một vài áp dụng ppt

Đang tải... (xem toàn văn)

Thông tin tài liệu

1 BẤT ĐẲNG THỨC MUIRHEAD MỘT VÀI ÁP DỤNG LÊ HỒ QUÝ, Trường THPT Duy Tân, Kon Tum Bất đẳng thức Muirheadmột dạng tổng quát rất quan trọng của bất đẳng thức AM-GM. Nó là một công cụ rất mạnh trong việc giải một số bài toán về bất đẳng thức. 1. Định lí Muirhead 1.1. Định nghĩa 1 (Bộ trội) Cho hai bộ số thực bất kì a = (a 1 , a 2 , , a n ) b = (b 1 , b 2 , , b n ). Ta nói bộ a trội hơn bộ b, kí hiệu a  b nếu chúng thỏa mãn các điều kiện sau đây i) a 1 ≥ a 2 ≥ ≥ a n và b 1 ≥ b 2 ≥ ≥ b n , ii) a 1 ≥ b 1 , a 1 + a 2 ≥ b 1 + b 2 , , a 1 + a 2 + + a n ≥ b 1 + b 2 + + b n và iii) a 1 + a 2 + + a n = b 1 + b 2 + + b n . 1.2. Định nghĩa 2 (Trung bình loại [a]) Giả sử x i > 0, 1 ≤ i ≤ n. Kí hiệu  !F (x 1 , x 2 , x n ) là tổng gồm n! biểu thức thu được từ F (x 1 , x 2 , x n ) bằng tất cả các hoán vị của x i . Ta sẽ chỉ xét trường hợp đặc biệt F (x 1 , x 2 , , x n ) = x α 1 1 x α 2 2 x α n n , với x i > 0, a i > 0. Khi đó trung bình loại [a] được định nghĩa bởi [a] = [a 1 , a 2 , , a n ] = 1 n!  !x α 1 1 x α 2 2 x α n n . Đặc biệt [1, 0, 0, , 0] = (n−1)! n! (x 1 + x 2 + + x n ) = 1 n n  i=1 x i là trung bình cộng của x i .  1 n , 1 n , , 1 n  = n! n!  x 1 n 1 .x 1 n 1 x 1 n 1  = n √ x 1 x 2 x n là trung bình nhân của x i . Khi a 1 + a 2 + + a n = 1 thì [a] là mở rộng thông thường của trung bình cộng trung bình nhân. 1.3. Định nghĩa 3 Gọi P (x, y, z) là một hàm ba biến x, y, z. Khi đó, ta định nghĩa i) Tổng hoán vị:  cyclic P (x, y, x) = P (x, y, z) + P (y, z, x) + P (z, x, y). ii) Tổng đối xứng:  sym P (x, y, x) = P (x, y, z) + P (x, z, y) + P (y, x, z) + P (y, z, x)+P (z, x, y)+P(z, y, x). www.VNMATH.com 2 1.4. Định lí Muirhead Định lí 1 (Bất đẳng thức Muirhead). Cho x i > 0, 1 ≤ i ≤ n a, b là hai bộ n số thực. Nếu a  b thì [a] ≥ [b]. Đẳng thức xảy ra khi chỉ khi a = b x 1 = x 2 = = x n . Chứng minh. Có thể xem phần chứng minh định lí Muirhead trong các tài liệu tham khảo [1], [2], [3], [4]. Vì rằng (1, 0, , 0)   1 n , 1 n , , 1 n  , nên bất đằng thức AM-GM là một hệ quả của bất đẳng thức Muirhead. 2. Một vài áp dụng Ở phần tiếp theo, chúng tôi xin trình bày một số áp dụng của bất đẳng thức Muirhead trong việc chứng minh bất đẳng thức. 2.1 Chứng minh các bất đẳng thức đại số Ví dụ 1. Cho ba số thực dương a, b, c. Chứng minh rằng (a + b)(b + c)(c + a) ≥ 8abc. Lời giải. Khai triển rút gọn ta được bất đẳng thức tương đương a 2 b + a 2 c + b 2 c + b 2 a + c 2 a + c 2 b ≥ 6abc. Vì (2, 1, 0)  (1, 1, 1) nên theo bất đẳng thức Muirhead ta có [(2, 1, 0)] ≥ [(1, 1, 1)]. Đẳng thức xảy ra khi chỉ khi a = b = c.  Ví dụ 2 (Yogoslavia-1991). Chứng minh rằng với mọi số thực dương a, b, c, ta luôn có 1 a 3 + b 3 + abc + 1 b 3 + c 3 + abc + 1 c 3 + a 3 + abc ≤ 1 abc . Lời giải. Quy đồng bỏ mẫu, rồi nhân hai vế cho 2, ta được bất đẳng thức tương đương  sym (a 3 + b 3 + abc)(b 3 + c 3 + abc) ≤ 2(a 3 + b 3 + abc)(b 3 + c 3 + abc)(c 3 + a 3 + abc) ⇔  sym (a 7 bc + 3a 4 b 4 c + 4a 5 b 2 c 2 + a 3 b 3 c 3 ) ≤  sym (a 3 b 3 c 3 + 2a 6 b 3 + 3a 4 b 4 c + a 7 bc + 2a 5 b 2 c 2 ) ⇔  sym (2a 6 b 3 − 2a 5 b 2 c) ≥ 0 Vì (6, 3, 0)  (5, 2, 2) nên theo bất đẳng thức Muirhead nên vế trái của bất đẳng thức cuối cùng là một hạng tử không âm. Từ đó, ta có điều phải chứng minh.  Nhận xét 1. Ở ví dụ tiếp theo, chúng ta sẽ sử dụng đến một kỹ thuật rất hữu ích sau đây www.VNMATH.com 3 Khi x 1 .x 2 x n = 1 thì [(a 1 , a 2 , , a n )] = [(a 1 − r, a 2 − r, , a n − r)], với r ∈ R. Ví dụ 3 (IMO-1995). Cho a, b, c là các số thực dương thỏa mãn điều kiện abc = 1. Chứng minh rằng 1 a 3 (b + c) + 1 b 3 (c + a) + 1 c 3 (a + b) ≥ 3 2 . Lời giải 1. Quy đồng bỏ mẫu, ta được bất đẳng thức tương đương 2(a 4 b 4 + b 4 c 4 + c 4 a 4 ) + 2(a 4 b 3 c + a 4 c 3 b + b 4 a 3 c + c 4 a 3 b + c 4 b 3 a) + 2(a 3 b 3 c 2 + b 3 c 3 a 2 + c 3 a 3 b 2 ) ≥ 3(a 5 b 4 c 3 + a 5 c 4 b 3 + b 5 c 4 a 3 + b 5 a 4 c 3 + c 5 a 4 b 3 + c 5 b 4 a 3 ) + 6a 4 b 4 c 4 . Sử dụng kí hiệu [a], ta được bất đẳng thức tương đương [(4, 4, 0)] + 2[(4, 3, 1)] + [(3, 3, 2)] ≥ 3[(5, 4, 3)] + [(4, 4, 4)] Để ý rằng 4 + 4 + 0 = 4 + 3 + 1 = 3 + 3 + 2 = 8, nhưng 5 + 4 + 3 = 4 + 4 + 4 = 12. Bởi vậy, ta có thể chọn r = 4 3 và sử dụng kỹ thuật trên ta được [(5, 4, 3)] =  ( 11 3 , 8 3 , 5 3 )  . Hơn nữa [(4, 4, 4)] =  ( 8 3 , 8 3 , 8 3 )  . Áp dụng bất đẳng thức Muirhead cho ba bộ số (4, 4, 0)   11 3 , 8 3 , 5 3  , (4, 3, 1)   11 3 , 8 3 , 5 3  , (3, 3, 2)   8 3 , 8 3 , 8 3  và cộng các bất đẳng thức vừa nhận được ta có bất đẳng thức phải chứng minh.  Lời giải 2. Bất đẳng thức đã cho tương đương với 1 a 3 (b + c) + 1 b 3 (c + a) + 1 c 3 (a + b) ≥ 3 2(abc) 4 3 Đặt a = x 3 , b = y 3 , c = z 3 , với x, y, z > 0. Khi đó bất đẳng thức trở thành  cyclic 1 x 9 (y 3 + z 3 ) ≥ 3 2x 4 y 4 z 4 . Quy đồng mẫu số chung bỏ mẫu, ta được bất đẳng thức  sym x 12 y 12 + 2  sym x 12 y 9 z 3 +  sym x 9 y 9 z 6 ≥ 3  sym x 11 y 8 z 5 +  sym x 8 y 8 z 8 hay   sym x 12 y 12 −  sym x 11 y 8 z 5  +2   sym x 12 y 9 z 3 −  sym x 11 y 8 z 5  +   sym x 9 y 9 z 6 −  sym x 8 y 8 z 8  ≥ 0. Vì (12, 12, 0)  (11, 8, 5), (12, 9, 3)  (11, 8, 5), (9, 9, 6)  (8, 8, 8) nên theo bất đẳng thức Muirhead thì mỗi hạng tử trong bất đẳng thức cuối là không âm. Từ đó ta có điều phải chứng minh.  www.VNMATH.com 4 Ví dụ 4 (IMO Shortlist-1998). Cho các số thực dương x, y, z thỏa mãn điều kiện xyz = 1. Chứng minh rằng x 3 (1 + y)(1 + z) + y 3 (1 + z)(1 + x) + z 3 (1 + x)(1 + y) ≥ 3 4 . Lời giải. Quy đồng mẫu số chung bỏ mẫu, ta được bất đẳng thức tương đương 4(x 4 + y 4 + z 4 + x 3 + y 3 + z ) ≥ 3(1 + x + y + z + xy + yz + zx + xyz). Sử dụng kí hiệu [a], ta được bất đẳng thức tương đương 4[(4, 0, 0)] + 4[(3, 0, 0)] ≥ [(0, 0, 0)] + 3[(1, 0, 0)] + 3[1, 1, 0] + [(1, 1, 1)]. Áp dụng bất đẳng thức Muirhead kỹ thuật trên, ta được [(4, 0, 0)] ≥  4 3 , 4 3 , 4 3  = [(0, 0, 0)] 3[(4, 0, 0)] ≥ 3[(2, 1, 1)] = 3[(1, 0, 0)] 3[(3, 0, 0)] ≥  4 3 , 4 3 , 1 3  = 3[(1, 1, 0)] và [(3, 0, 0)] ≥ [(1, 1, 1)]. Cộng các bất đẳng thức vừa nhận được ta có bất đẳng thức phải chứng minh. Đẳng thức xảy ra khi chỉ khi x = y = z = 1.  Nhận xét 2. i) Trong bài toán trên, ta có thể "nới" bớt điều kiện ràng buộc thành điều kiện rộng hơn xyz ≥ 1. Khi đó, ta có bất đẳng thức [(a 1 , a 2 , a 3 )] ≥ [(a 1 − r, a 2 − r, a 3 − r)], trong đó r ≥ 0 bất kì. ii) Sử dụng bất đẳng thức AM-GM, ta dễ dàng chứng minh được kết quả sau: Với hai bộ n số thực a b, ta luôn có [a] + [b] 2 ≥  a + b 2  . Ví dụ 5 (IMO-2005). Cho x, y, z là các số thực dương sao cho xyz ≥ 1. Chứng minh rằng x 5 − x 2 x 5 + y 2 + z 2 + y 5 − y 2 y 5 + z 2 + x 2 + z 5 − z 2 z 5 + x 2 + y 2 ≥ 0. Lời giải. Sau khi quy đồng mẫu số chung, bỏ mẫu sử dụng kí hiệu [a], ta được bất đẳng thức tương đương [(9, 0, 0)]+4[7, 5, 0]+[(5, 2, 2)]+[(5, 5, 5)] ≥ [(6, 0, 0)+[(5, 5, 2)]+2[(5, 4, 0)]+2[(4, 2, 0)]+[(2, 2, 2)]. www.VNMATH.com 5 Ta có (1) [9, 0, 0] ≥ [(7, 1, 1)] ≥ [(6, 0, 0)] (do bất đẳng thức Muirhead nhận xét 2.i)) (2) [(7, 5, 0)] ≥ [(5, 5, 2)] (do bất đẳng thức Muirhead) (3) 2[(7, 5, 0)] ≥ 2[(6, 5, 1)] ≥ 2[(5, 4, 0)] (do bất đẳng thức Muirhead nhận xét 2.i)) (4) [(7, 5, 0)] + [(5, 2, 2)] ≥ 2  6, 7 2 , 1  ≥ 2  11 2 , 7 2 , 3 2  ≥ 2[(4, 2, 0)] (do bất đẳng thức Muirhead nhận xét 2) (5) [(5, 5, 5)] ≥ [(2, 2, 2)] (do nhận xét 2.i)) Cộng các bất đẳng thức trên vế với vế, ta được bất đẳng thức cần chứng minh.  2.2 Chứng minh các bất đẳng thức hình học Ví dụ 6 (IMO-1961). Cho a, b, c là độ dài ba cạnh của tam giác AB C, S là diện tích của tam giác đó. Chứng minh rằng 4 √ 3S ≤ a 2 + b 2 + c 2 . Lời giải 1. Sử dụng công thức Heron, ta có thể viết lại bất đẳng thức đã cho như sau a 2 + b 2 + c 2 ≥ 4 √ 3  (a + b + c) 2 (a + b − c) 2 (a + c − b) 2 (b + c − a) 2 . Bình phương cả hai vế của bất đẳng thức, ta được ⇔ (a 2 + b 2 + c 2 ) 2 ≥ 3[(a + b) 2 − c 2 )(c 2 − (b − a) 2 )] = = 3(2c 2 a 2 + 2c 2 b 2 + 2a 2 b 2 − (a 4 + b 4 + c 4 )) ⇔ a 4 + b 4 + c 4 ≥ a 2 b 2 + b 2 c 2 + c 2 a 2 Dùng kí hiệu [a], ta có bất đẳng thức tương đương [(4, 0, 0)] ≥ [(2, 2, 0)] Bất đẳng thức cuối cùng này luôn đúng do bất đẳng thức Muirhead ứng với hai bộ số (4, 0, 0)  (2, 2, 0).  Lời giải 2. Đặt x = a + b − c, y = c + a − b, z = b + c − a, ta thu được x + y + z = a + b + c, khi đó, dùng công thức Heron ta có 4S =  (a + b + c)(xyz) ≤  (a + b + c) (x + y + z) 3 27 = (a + b + c) 2 3 √ 3 . Lúc này, ta chỉ cần chứng minh (a + b + c) 2 ≤ 3(a 2 + b 2 + c 2 ). Bất đẳng thức cuối cùng này được suy ra từ bất đẳng thức Muirhead, vì rằng [(1, 1, 0)] ≤ [(2, 0, 0)].  Ví dụ 7. Xét tam giác ABC với độ dài các cạnh là a, b, c R, r lần lượt là bán kính của đường tròn ngoại tiếp, nội tiếp của tam giác đó. Chứng minh rằng r R ≤  2[2a 2 − (b − c) 2 ][2b 2 − (c − a) 2 ][2c 2 − (a − b) 2 ] (a + b)(b + c)(c + a) . www.VNMATH.com 6 Lời giải. Trước hết, ta thuần nhất bất đẳng thức phải chứng minh với các biến x, y, z bằng cách dùng các đồng nhất thức R = abc 4S , r = S p , S 2 = p(p − a)(p − b)(p −c), p = a + b + c 2 và đặt a = y + z 2 , b = z + x 2 , c = x + y 2 . Khi đó, ta có x = b + c − a > 0, y = c + a − b > 0, z = a + b − c > 0. Bình phương hai vế rút gọn ta được bất đẳng thức tương đương 105[(4, 4, 4)] + 264[(5, 4, 3)] + 88[(6, 3, 3)] + 48[(7, 3, 2)] + 9[(8, 2, 2)] ≤ ≤ 136[(5, 5, 2)] + 106[(6, 4, 2)] + 176[(6, 5, 1)] + 7[(6, 6, 0)] + 72[(7, 4, 1)]+ + 8[(7, 5, 0)] + 8[(8, 3, 1)] + [(8, 4, 0)]. Áp dụng bất đẳng thức Muirhead, ta lần lượt có 9[(8, 2, 2)] ≤ 8[(8, 3, 1)] + [(8, 4, 0)] 48[(7, 3, 2)] ≤ 48[(7, 4, 1)] 88[(6, 3, 3)] ≤ 88[(6, 5, 1)] 264[(5, 4, 3)] ≤ 136[(5, 5, 2)] + 106[(6, 4, 2)] + 22[(6, 5, 1)] 105[(4, 4, 4)] ≤ 66[(6, 5, 1)] + 7[(6, 6, 0)] + 24[(7, 4, 1)] + 8[(7, 5, 0)]. Cộng các vế tương ứng của các bất đẳng thức trên, ta được bất đẳng thức cần chứng minh. Đẳng thức xảy ra khi chỉ khi x = y = z hay tam giác ABC đều.  Bài tập Bài 1. Chứng minh rằng mọi số thực dương a, b, c, ta luôn có a 5 + b 5 + c 5 ≥ a 3 bc + b 3 ca + c 3 ab. Bài 2. Cho a, b, c là các số thực dương. Chứng minh rằng a (a + b)(a + c) + b (b + c)(b + a) + c (c + a)(c + b) ≤ 9 4(a + b + c) . Bài 3 (IMO-1964). Cho a, b, c là các số thực dương. Chứng minh rằng a 3 + b 3 + c 3 + 3abc ≥ ab(a + b) + bc(b + c) + ca(c + a). Bài 4 (Iberoamerican Shortlist-2003). Cho ba số thực dương a, b, c. Chứng minh rằng a 3 b 2 − bc + c 2 + b 3 c 2 − ca + a 2 + c 3 a 2 − ab + b 2 ≥ a + b + c. www.VNMATH.com 7 Bài 5 (IMO-1984). Cho x, y, z là các số thực không âm thỏa mãn điều kiện x+y+z = 1. Chứng minh rằng 0 ≤ xy + yz + zx − 2xyz ≤ 7 27 . Bài 6. Cho x, y, z là các số thực không âm sao cho xy + yz + zx = 1. Chứng minh 1 x + y + 1 y + z + 1 z + x ≥ 5 2 . www.VNMATH.com 8 Tài liệu tham khảo [1] G. H. Hardy, J. E. Littlewood, G. Polya, Inequalities, Cambridge University Pree, 1967. [2] Radmila Bulajich Manfrino, José Antonio Gosmez Ortega, Rogelio valdez Delgado, Inequalities, A Mathematical Olympial Approach, Birkh¨auser, 2009. [3] Lau Chi Hin, Muirhead’s Inequality, Vo.11. Mathematical Excalibur, 2006. [4] Ivan Matié, Classical Inequalities, Olympial Training Materials, 2007. [5] Zoran Kadelburg, Du˜san Đukié, Milivoje Lukié, Ivan Matié, Inequalities of Karamata, Schur and Muirhead, and some applications, The teaching of Mathematics, Vol. VIII, pp. 31-45, 2005. [6] Cezar Lupu, Tudorel Lupu, Problem 11245, American mathematical Monthly, Vol.113, 2006. [7] Andre Rzym, Muirhead’s Inequality, November 2005. [8] Stanley Rabinowitz, On The Computer Solution of Symmetric Homogeneous Triangle Inequalities, Alliant Computer Systems Corporation Littleton, MA 01460. [9] Nguyễn Văn Mậu, Bất đẳng thức, Định lý áp dụng, NXB Giáo Dục, 2005. [10] Phạm Kim Hùng, Sáng tạo bất đẳng thức, NXB Hà Nội, 2007. [11] Ngô Thế Phiệt, Một số phương pháp mới trong chứng minh bất đẳng thức, NXB Giáo Dục, 2007. www.VNMATH.com . 1 BẤT ĐẲNG THỨC MUIRHEAD VÀ MỘT VÀI ÁP DỤNG LÊ HỒ QUÝ, Trường THPT Duy Tân, Kon Tum Bất đẳng thức Muirhead là một dạng tổng quát rất quan trọng của bất đẳng thức. nên bất đằng thức AM-GM là một hệ quả của bất đẳng thức Muirhead. 2. Một vài áp dụng Ở phần tiếp theo, chúng tôi xin trình bày một số áp dụng của bất đẳng

Ngày đăng: 19/02/2014, 21:20

Từ khóa liên quan

Mục lục

  • www.VNMATH.com

Tài liệu cùng người dùng

  • Đang cập nhật ...

Tài liệu liên quan